Solving Harmonic Motion: Find Equilibrium Points & Frequency

In summary: Do you think a larger mass would have a higher frequency? You might want to check ##2k_1## as...I don't know, but it's worth checking.
  • #1
physics148
13
0

Homework Statement


A potential energy function for a particle moving in one-dimension is given as:
V (x) =k1x^2/(2)+k2/x

(a) Locate all the equilibrium points.
(b) Show that the motion is always periodic for any amount of total energy.
(c) What is the frequency f the motion if the amplitude of oscillation is very small?

Homework Equations



The Attempt at a Solution

:[/B]
I figured out part a, and I couldn't figure out how to do c. Any advice on how to do it would be appreciated.
My explanation for b was as follows:
since the second derivative of the potential is positive, the particle will undergo periodic motion since no amount of energy can make it "escape", rather, it will be trapped forever and undergo periodic motion since the potential energy graph is positive.

Is this a correct explanation?
 
Last edited by a moderator:
Physics news on Phys.org
  • #2
(a) equilibrium points are the stationary points of V(x) right - they are stable if the point is concave up. This what you said?
(b) V(x) is concave up everywhere in x, therefore there is no energy higher than V. This is basically what you said.
(c) You have to use the definition of periodic motion, frequency etc. ... maybe the equation of motion will help here? F = -V' = ma
 
  • #3
Simon Bridge said:
(a) equilibrium points are the stationary points of V(x) right - they are stable if the point is concave up. This what you said?
(b) V(x) is concave up everywhere in x, therefore there is no energy higher than V. This is basically what you said.
(c) You have to use the definition of periodic motion, frequency etc. ... maybe the equation of motion will help here? F = -V' = ma
Yup that's what I said for a and b. Not sure how F=-V' will help for part c? What formula do I use, and how do I use the fact that the amplitude is extremely small?

Thanks
 
  • #4
physics148 said:
Yup that's what I said for a and b. Not sure how F=-V' will help for part c? What formula do I use, and how do I use the fact that the amplitude is extremely small?

Thanks

You need to look at how ##F## varies about the equilibrium point, ##x_0##. Finding a Taylor series expansion often helps. Does that make any sense?
 
  • #5
PeroK said:
You need to look at how ##F## varies about the equilibrium point, ##x_0##. Finding a Taylor series expansion often helps. Does that make any sense?
I was thinking of using a taylor series expansion, but I'm unsure on how I would go about doing this? I"m not very proficient with taylor series, could you help me set it up?
 
  • #6
physics148 said:
I was thinking of using a taylor series expansion, but I'm unsure on how I would go about doing this? I"m not very proficient with taylor series, could you help me set it up?

Let me explain the idea. The Taylor series For ##F## about ##x_0## is:

##F(x) = F(x_0) + F'(x_0)(x-x_0) + \frac12 F''(x_0) (x-x_0)^2 \dots ##

Now, if ##(x-x_0)## is small, then you can neglect the terms in ##(x-x_0)^2## and higher. Further, if ##F(x_0) = 0## (an equilibrium point), then:

##F(x) \approx F'(x_0)(x-x_0)##

And, if ##x_0## is a local minimum for ##V##, then ##V''(x_0) = -F'(x_0) > 0##

So, we have an equation for simple harmonic motion:

##m\ddot{x} = -k(x-x_0)##, for some ##k = -F'(x_0) > 0##
 
  • #7
PeroK said:
Let me explain the idea. The Taylor series For ##F## about ##x_0## is:

##F(x) = F(x_0) + F'(x_0)(x-x_0) + \frac12 F''(x_0) (x-x_0)^2 \dots ##

Now, if ##(x-x_0)## is small, then you can neglect the terms in ##(x-x_0)^2## and higher. Further, if ##F(x_0) = 0## (an equilibrium point), then:

##F(x) \approx F'(x_0)(x-x_0)##

And, if ##x_0## is a local minimum for ##V##, then ##V''(x_0) = -F'(x_0) > 0##

So, we have an equation for simple harmonic motion:

##m\ddot{x} = -k(x-x_0)##, for some ##k = -F'(x_0) > 0##
Okay, thanks. Here is what we have so far, but we are getting stuck when trying to solve for the frequency, since we get it as an implicit function? For reference, our local min ##x_0=(\frac{k_2}{k_1})^{\frac13}##

upload_2016-10-30_13-8-43.png
 
  • #8
physics148 said:
Okay, thanks. Here is what we have so far, but we are getting stuck when trying to solve for the frequency, since we get it as an implicit function? For reference, our local min ##x_0=(\frac{k_2}{k_1})^{\frac13}##

View attachment 108218
I can't read that. The frequency is always the same, based on ##k/m##
 
  • #9
PeroK said:
I can't read that. The frequency is always the same, based on ##k/m##

So, we solved for k, and got it to be equal to ##2k_1##. So the corresponding period then would just be ##2 \pi \sqrt{\frac{2k_1}{m}}## and the frequency is just ##\frac{1}{2\pi}\sqrt{\frac{m}{2k_1}}##?
 
  • #10
physics148 said:
So, we solved for k, and got it to be equal to ##2k_1##. So the corresponding period then would just be ##2 \pi \sqrt{\frac{2k_1}{m}}## and the frequency is just ##\frac{1}{2\pi}\sqrt{\frac{m}{2k_1}}##?
Do you think a larger mass would have a higher frequency? You might want to check ##2k_1## as well.
 
  • #11
PeroK said:
Do you think a larger mass would have a higher frequency? You might want to check ##2k_1## as well.
Sorry, I got the inside of of roots mixed up.

Based on the taylor series expansion, we get ##F(x_0) = -k(x-x_0)##, where ##k=-F(x_0)##. Then for an initial point ##x_0=(\frac{k_2}{k_1})^{\frac 13}## ##k=-(k_1+\frac{k_2}{x^3})=-(k_1+k_2\frac{k_2}{(\frac{k_2}{k_1}^3)^{\frac 13}}=-2k_1##. The problem that we run into here, is that to solve for the angular frequency ##\omega_0## in the equation ##f=\frac{\omega_0}{2\pi}## using equations that we have derived in other parts of our assignment and in class, ##x(t)=Asin(\omega t)## where A is the amplitude, we get an implicit function which appears to be unsolvable due to it being ##-k(x-x_0)## rather than ##-kx## which is the typical function for a spring. If we do sub k into the standard equations for a spring, we end up getting a negative within a square root, which gives an imaginary frequency.

Could you possibly point us in the right direction from here?
 
  • #12
physics148 said:
Sorry, I got the inside of of roots mixed up.

Based on the taylor series expansion, we get ##F(x_0) = -k(x-x_0)##, where ##k=-F(x_0)##. Then for an initial point ##x_0=(\frac{k_2}{k_1})^{\frac 13}## ##k=-(k_1+\frac{k_2}{x^3})=-(k_1+k_2\frac{k_2}{(\frac{k_2}{k_1}^3)^{\frac 13}}=-2k_1##. The problem that we run into here, is that to solve for the angular frequency ##\omega_0## in the equation ##f=\frac{\omega_0}{2\pi}## using equations that we have derived in other parts of our assignment and in class, ##x(t)=Asin(\omega t)## where A is the amplitude, we get an implicit function which appears to be unsolvable due to it being ##-k(x-x_0)## rather than ##-kx## which is the typical function for a spring. If we do sub k into the standard equations for a spring, we end up getting a negative within a square root, which gives an imaginary frequency.

Could you possibly point us in the right direction from here?
Sorry, our ##k## should be ##3k_1##, not ##-2k_1##. That would give us a real number for the frequency, but I'm still not sure if it makes sense based on the derivation for the angular frequency that we did in class
 
  • #13
physics148 said:
Sorry, I got the inside of of roots mixed up.

Based on the taylor series expansion, we get ##F(x_0) = -k(x-x_0)##, where ##k=-F(x_0)##. Then for an initial point ##x_0=(\frac{k_2}{k_1})^{\frac 13}## ##k=-(k_1+\frac{k_2}{x^3})=-(k_1+k_2\frac{k_2}{(\frac{k_2}{k_1}^3)^{\frac 13}}=-2k_1##. The problem that we run into here, is that to solve for the angular frequency ##\omega_0## in the equation ##f=\frac{\omega_0}{2\pi}## using equations that we have derived in other parts of our assignment and in class, ##x(t)=Asin(\omega t)## where A is the amplitude, we get an implicit function which appears to be unsolvable due to it being ##-k(x-x_0)## rather than ##-kx## which is the typical function for a spring. If we do sub k into the standard equations for a spring, we end up getting a negative within a square root, which gives an imaginary frequency.

Could you possibly point us in the right direction from here?

You need to check ##F'(x)##, especially the derivative of ##1/x^2##.

Second, there's no real difference having ##x-x_0##, as that just adds a nonzero equilibrium point. You could change variables to ##u = x - x_0## if you're really stuck.

For positive ##k##, you get ##\sin## and ##\cos## solutions to your differential equation.
 
  • #14
physics148 said:
Sorry, our ##k## should be ##3k_1##, not ##-2k_1##. That would give us a real number for the frequency, but I'm still not sure if it makes sense based on the derivation for the angular frequency that we did in class
Yes, ##3k_1## is correct. It's just a positive constant, so why would it not make sense. The period and frequency are as you had them before, although you has them inverted.
 
  • #15
Thank you, I fixed my errors and everything worked out
 

1. What is harmonic motion?

Harmonic motion is a type of periodic motion in which a system or object oscillates about an equilibrium point with a constant frequency and amplitude. This type of motion can be described by a sine or cosine function.

2. How do you find equilibrium points in harmonic motion?

To find equilibrium points in harmonic motion, you need to set the acceleration of the system equal to zero. This will give you the points at which the system is in equilibrium, meaning there is no net force acting on the system.

3. What is the formula for calculating frequency in harmonic motion?

The formula for calculating frequency in harmonic motion is f = 1 / T, where f is the frequency and T is the period of the motion. The period can be found by taking the inverse of the frequency, or by calculating the time it takes for the system to complete one full cycle of motion.

4. How can I determine the amplitude of harmonic motion?

The amplitude of harmonic motion can be determined by measuring the maximum displacement from the equilibrium point. It is usually represented by the letter A and is equal to half of the distance between the highest and lowest points of the oscillation.

5. Can harmonic motion be applied to real-life situations?

Yes, harmonic motion can be observed in many real-life situations such as the motion of a pendulum, a mass-spring system, or sound waves. It is a fundamental concept in physics and is used to model and understand various natural phenomena.

Similar threads

  • Introductory Physics Homework Help
2
Replies
51
Views
2K
  • Introductory Physics Homework Help
Replies
16
Views
401
Replies
13
Views
311
Replies
2
Views
1K
  • Advanced Physics Homework Help
Replies
10
Views
2K
  • Advanced Physics Homework Help
Replies
5
Views
1K
  • Advanced Physics Homework Help
Replies
9
Views
5K
  • Introductory Physics Homework Help
Replies
10
Views
2K
  • Advanced Physics Homework Help
Replies
3
Views
2K
  • Introductory Physics Homework Help
Replies
1
Views
2K
Back
Top